TExES Generalist Ec-6 191 Exam (Practice Test I)

Reviewed by Editorial Team
The ProProfs editorial team is comprised of experienced subject matter experts. They've collectively created over 10,000 quizzes and lessons, serving over 100 million users. Our team includes in-house content moderators and subject matter experts, as well as a global network of rigorously trained contributors. All adhere to our comprehensive editorial guidelines, ensuring the delivery of high-quality content.
Learn about Our Editorial Process
| By Falmendarez
F
Falmendarez
Community Contributor
Quizzes Created: 3 | Total Attempts: 2,952
| Attempts: 2,397 | Questions: 140
Please wait...
Question 1 / 140
0 %
0/100
Score 0/100
1. Oftentimes the use of acronyms facilitates the learning process. What is a common acronym for remembering the order of operations?

Explanation

In mathematics, there is a specific order in which actions must occur in order to successfully reach the correct answer. The guideline used to remember this order is typically stated as Please Excuse My Dear Aunt Sally (PEMDAS), which stands for Parentheses Exponent Multiply Divide Add Subtract. Competency 014

Submit
Please wait...
About This Quiz
TExES Generalist Ec-6 191 Exam (Practice Test I) - Quiz

This practice test for the TExES Generalist EC-6 191 exam assesses knowledge across multiple subjects including English language arts, mathematics, and reading strategies. It's designed to prepare candidates... see morefor certification, focusing on essential academic skills and teaching strategies. see less

2. Traditionally, matter has been classified into three states

Explanation

Matter can change from liquid to solid to gas in a cyclical fashion. (A) is incorrect because all three represent examples of solid matter. (B) is incorrect because it presents only examples of the states and fails to address the stages represented. (D) is incorrect because it presents just examples of solid and liquid matter. Competency 031

Submit
3. Newspapers and magazines have traditionally been considered what type of media?

Explanation

Newspapers and magazines have been considered print media. (B) is incorrect because images are often used to complement texts being displayed in such mediums. (A) is incorrect because there is no such a thing as "electric" media; the correct term is "electronic" media. Competency 011

Submit
4. By second grade, students are generally guided to discontinue the practice of pointing to the words being read. What is the rationale for this change of strategy?

Explanation

Pointing to the words as they are read is designed to establish the connection between speech and print. However, after the skill has been mastered, it is discontinued so students can engage in fluent reading. (A) and (B) are incorrect because they offer non-substantial reasons for the strategy change. (D) is incorrect because the practice of pointing to the words as they are being read is not archaic; it is indeed an effective practice for beginning readers. Competency 005

Submit
5. Identify the steps that best describe the writing process.

Explanation

Brainstorming for ideas and developing an outline of the writing project are paramount for process writing. (A), (B), and (D) are incorrect because they do not contain brainstorming as the initial component of the process. Competency 008

Submit
6. What causes earthquakes?

Explanation

The movement of the plates causes intense seismological activity near the faults, resulting in earthquakes. (A) and (C) have some merit because thermal activity can have some implications, but these choices do not provide specific answers to the question. (D) is totally incorrect; there is no known connection between the rotation of the Earth and earthquakes. Competency 039

Submit
7. Based on the Individuals with Disabilities Education Act (IDEA), children with crutches or any other types of assistive devices should

Explanation

The ADA required school districts to place special education children in the least restrictive educational environment; thus, an orthopedic handicapped student would be placed in mainstream classrooms. Students are required to receive physical education using appropriate accommodations to allow their successful participation. (A) is incorrect because only in extreme cases are special education children excluded from participating in the school curriculum. (B) is partially correct, but it fails to mention the need to make accommodations for successful participation in physical education activities. (D) is incorrect because it adopts an extreme position not applicable to limited mobility. Competency 045

Submit
8. Identify the factor(s) that most likely can affect the development of early literacy among preschool children.

Explanation

Reading to children and having books available for them provide children with reading readiness skills and can promote interest in reading. (B) is incorrect because the use of dictionary skills is not developmentally appropriate for preschool children. (C) is incorrect because the children might not be ready for electronic translation programs, and translation is not the best strategy for promoting literacy development. (D) is incorrect because getting books for children without additional support will not develop early literacy among children. Competency 004

Submit
9. The most appropriate exercises for kindergarten and first-grade students are

Explanation

Fun games involving physical activity are the best way to exercise children in kindergarten. (A) is incorrect because, traditionally, jogging for fitness might be inappropriate for children at this early age. (B) and (C) are incorrect because contact sports and activities that require skill and/or endurance are not developmentally appropriate for kindergarten or first-grade students. Competency 045

Submit
10. The key function of the food chain in the ecosystem is to

Explanation

An ecosystem needs to have a balance among organisms to ensure the ability of the system to support life. (A), (B), and (D) are incorrect because they merely represent examples of how the system maintains a balance. Competency 027

Submit
11. The economy theory that states that prices vary based on balance between the availability of a product or service at a certain price and the desire of potential purchasers to pay that price is known as

Explanation

The economic theory that states that prices vary based on a balance between the availability of a product or service at a certain price and the desire of potential purchasers to pay that price is known as supply and demand. Free enterprise (A) is a generic term to describe capitalism and entrepreneurship. Inflation and deflation (C) are general terms to describe the price stability in the nation and the purchasing power of consumers. Competency 022

Submit
12. To make the science curriculum cognitively accessible to English language learners in grades K–2, teachers use thematic units and incorporate

Explanation

The state curriculum requires students to engage in scientific exploration in grades K–12. Hands-on experimentation promotes active learning and is beneficial to all students, but it is especially important and developmentally appropriate for children in grades K–2. Choice (A) is incorrect because the incorporation of foreign language methods is a very broad statement, and it does not say specifically which method will be used or how it will be implemented. (C) is also incorrect because the statement is vague and does not describe the types of sophisticated tools. Traditionally, in K–2, the tools used are very simple and concrete. (D) is incorrect because the state curriculum does not require the use of chemicals and human specimens in the early grades. Competency 028

Submit
13. What are Venn diagrams commonly used for in reading instruction?

Explanation

Venn diagrams can be used for multiple purposes in education, but in language arts it is commonly used to compare similarities, differences, and common elements between two stories. Traditionally, Venn diagrams are not used to identify the parts of the story (B), nor to identify the main idea of a story (D). Semantic mappings (C), not Venn diagrams, are commonly used to identify key attributes of a given concept. Competency 004

Submit
14. Mrs. Morris presented a lesson to her fifth graders about the early development of major science concepts. She discussed how people in Europe thought that the Earth was the center of the universe at a given time in history, and how first physicians bled the patients to try to cure them. Other than the historical value of the lesson, what is the educational purpose of this lesson?

Explanation

Students need to have a clear understanding of the evolution of science concepts, so they can develop an appreciation for the status of science today. (B) is incorrect because it addresses only one historical period, the Middle Ages. It also addresses the historical value of the information, which should be excluded from the answer. (C) is incorrect because it only addresses the medical implications of the information. (D) presents an incorrect statement, because at the present time most European countries have well-developed scientific knowledge. Competency 025

Submit
15. Identify the order of the products that have supported the Texas economy through its history.

Explanation

Cotton and eventually cattle were the main products produced in Texas during the nineteenth and early twentieth centuries. Later, with the discovery of petroleum in the twentieth century, it became one of the most important exports for Texas. In the latter part of the twentieth century and until today, computer and electronics replaced oil as the main products of the state. Based on this explanation, options (A), (B), and (D) are eliminated. Competency 022

Submit
16. Why do most countries in the world use the metric system in scientific research and engineering?

Explanation

The metric system is a very precise system that allows for the measurement of very small amounts of matter. For example, the metric system uses measurements divided into thousands (millimeters or milligrams), while the English system uses larger units of measurement, like inches and ounces. (A) and (B) contain perhaps true statements and opinions, but they fail to address the question. (D) is incorrect; the English system does not have a simple way to compute measures. The metric system is based on 10, and the computations are a lot easier. Competency 024

Submit
17. In addition to posting the appropriate rules for laboratory safety, teachers send home a safety contract for parents to sign. What is the purpose of sending this type of document home?

Explanation

The purpose of sending safety or any type of contracts for the parents' signature is to make parents aware of the guidelines and expected behaviors in school. Contracts of this type are developed for school purpose only. These are not traditionally used to guide parents to follow safety procedures at home (A), or to guide children to commit to the study of science (B). The purpose of sending the contract home is to make parents aware of the expected behavior, not necessarily to make parents responsible for the behavior of the children (D). Competency 024

Submit
18. The U.S. Constitution was designed so that no single branch of the government—executive, judicial, or legislative—could exert full control over the other two. This unique feature of the Constitution is known as

Explanation

The U.S. Constitution set up a system so that each branch has the power to control or regulate the power of the other two. (A) is incorrect because judicial review addresses only the power of the judicial branch. Judicial review is the power to review legislation enacted by Congress and signed by the president. If the legislation is unconstitutional, it can be invalidated. (B) is incorrect because it addresses the power of the president to veto or refuse to sign legislation approved by Congress. (D) is incorrect because habeas corpus is one of the civil rights that guarantees people the right to a quick trial by jury. Competency 023

Submit
19. During reading group activity, Ms. Lueck often asks her second graders to orally retell the main ideas of a story they have just read together. This is an example of which type of assessment?

Explanation

An oral retelling is an informal assessment that seeks to determine what a student can recall about a story. It might be scored using a checklist that the teacher created. (B) is incorrect because norm-reference tests compare a student's achievement to grade- or age-level peers, typically in a bell curve. (C) is incorrect because criterion-referenced tests are often more formal and have a target or uniform criteria that students must meet to "pass" the test. (D) is incorrect because an oral retelling is harder to score objectively and is thus typically not standardized. Competency 012

Submit
20. Mrs. Thompson has requested that their students create a PowerPoint presentation on the history of visual media. Before working on such a presentation, students need to pick a topic to create their presentation. Which of the following would be the most appropriate topic for such a presentation?

Explanation

The most appropriate topic would be for students to research the evolution of billboards on the highway (A). (B), (C), and (D) are incorrect because they represent examples of electronic media, not visual media. Competency 011

Submit
21. Scenario: Mr. Jetter organized a lesson to introduce sixth-grade students to the concept of marine habitats. As a focus activity he presented two 25-gallon aquariums. Aquarium A has a variety of aquatic plants, 10 snails, and 25 unique varieties of small fishes. Aquarium B contains a variety of aquatic plants, three snails, and five small fish of the same variety. To implement the lesson, he organized the students in groups of five. Once the groups were formed, students were asked to discuss the appropriateness of the combination of aquatic plants and number of animals. The ultimate goal is to predict the success of the habitats, including successful breeding among animals. What is the main science objective of Mr. Jetter's lesson?

Explanation

Using high-interest activities like observing live animals and plants in an aquarium, students can be guided to used scientific inquiry and problem-solving skills. Competency 035

Submit
22. What is the primary disadvantage of using written essay to assess the performance of English language learners (ELLs)?

Explanation

A main disadvantage is that ELLs may have difficulty expressing their thoughts in writing. This means the test may not be a valid way to measure student's knowledge about a particular topic. Option (A) is incorrect because many students and teachers are familiar with rubric-based scoring. Answers (B) and (D) are incorrect because they portray the advantages of using an essay to assess knowledge, rather than disadvantages. Competency 012

Submit
23. Students in Mr. Campos' classroom are learning to interpret visual images. As part of Mr. Campos' explanation to his class, he is considering mentioning all of the following benefits of visual media EXCEPT

Explanation

Visual media provides many benefits to students. Some of these include helping students' spark their audience's attention when they are trying to convey a message (A), adding clarity to their message (C) as they can be used to complement not only their texts but the information they are talking about, and allowing students to show step-by-step development ideas (D). Therefore, the only option that does not offer benefits of visual media is (B). Competency 011

Submit
24. Probably as a response to the war in Iraq and Afghanistan, the Organization of Petroleum Exporting Countries (OPEC) cut the production of oil. As a result of this action, the cost of gasoline increased to almost $3.00 per gallon in 2005. What is the economic principle or statement that is best represented by this situation?

Explanation

Reducing the production of oil while keeping the same demand for the product creates an imbalance between supply and demand. This imbalance results in a price increase. (A) is incorrect. (B) is incorrect because the statement does not explain the economic principle required in the question. Generally speaking, war increases the demand and prices of many things, not only the price of fossil fuels. (B) just presents a true statement—the American economy depends on foreign oil. (D) presents an opinion that fails to address the true question. Competency 022

Submit
25. One of the best known models of scientific inquiry is the 5-E model. In this model, students go through five steps: engage, explore, explain, elaborate, and evaluate. What is the main purpose of the "explore" component of the process?

Explanation

In the "explore" component of the 5-E Model, students begin to have hands-on exposure to the topic. In this stage, students are guided to make observations and gather data about the scientific project. (B) is incorrect because it describes the initial stage of the process, "engage," in which students begin to conceptualize the topic of the study. (C) describes the process of concept invention or explanation, through which a teacher-guided discussion presents the findings of the study. (D) is incorrect, because it describes the "engage" step of the process, in which students are motivated to learn about the scientific concept. Competency 025

Submit
26. The imperial forces of Japan attacked Pearl Harbor on Sunday, December 7, 1941. Once the attack began, the officer in charge called President Roosevelt and notified him of the attack. During the communication, the president was able to hear the noise of the bombs exploding and the struggle of the battle. Recently, a reporter from the Dallas Morning News was researching the details of the attack and found a transcription of the official diary of Emperor Hirohito, where he indicated the following: "The Imperial forces of Japan destroyed the American fleet in Hawaii Monday, December 8, 1941." If the attack occurred on December 7, why did the official document from the Japanese government indicate that the attack happened on December 8? Select the statement that best explains this discrepancy.

Explanation

There is a difference of one day from Honolulu to Tokyo. Honolulu is located west of the International Date Line (IDL), while Japan is east of the line. That is, if we travel from Honolulu toward the east on December 7 and pass the IDL to get to Tokyo, we would get there on December 8. The attack was recorded on December 7, 1941, in Honolulu, but the equivalent date for Japan was December 8, 1941. (A), (B), and (D) are incorrect because they do not explain the real reason for the date confusion. Additionally, the scenario does not provide information to support the position that Emperor Hirohito was confused about the dates. Competency 021

Submit
27. Identify the set of words representing the concept of antonyms

Explanation

Antonyms are words that indicate "opposites." The only pair that does so is (B), small-large. The set of words in (A), small and the comparative, smaller, represent two different words. The words in (C), bear—bear, represent an example of homonyms—words with the same pronunciation and spelling, but with different meanings. (D) represents an example of synonyms, words with equivalent meaning. Competency 002

Submit
28. In the problem, , what is the first operation which should be performed according to the order of operations?

Explanation

Using the acronym PEMDAS as a mnemonic device to remember the order of operations, it allows us to see that the first operation required in the problems is multiplication. That is, the acronym calls for the following order: percentages, exponents, multiplication, division, addition, and subtraction. However, since the problem does not contain percentages or exponents, the first operation required in the problem is multiplication. Competency 018

Submit
29. The main purpose for teaching sight words is to promote instant recognition of characters and words. Instant recognition of words can improve students'

Explanation

When children don't have to struggle to decode words, reading fluency is facilitated. (B) is incorrect because, by definition, no decoding is necessary when the reader recognizes the word instantly. (C) is incorrect because comprehension goes beyond the instant recognition of words. Readers have to analyze the schema of the writing and evaluate other elements to achieve reading comprehension. Instant recognition of words can improve spelling skills; however, it does not necessarily improve the writing process (D). Competency 006

Submit
30. The process by which physical movements develop and become specialized for motor performance depends primarily upon

Explanation

Repetition and practice of a movement pattern is the fastest way to master a physical skill no matter the age level. (A) has no relevance to skill acquisition. (B) and (D) could in part attribute to skill development, although not to the same level of guided repetition and practice. Competency 045

Submit
31. Identify the mathematical properties involved in the following problem: (6 + 2) + 5 = 6 + (2 + 5).

Explanation

One of the basic properties of addition is that it can be carried out in any order. Therefore, since addition is the only operation that is performed in this exercise, one knows the correct answer is the associate property of addition. Competency 013

Submit
32. Scenario: Mr. Jetter organized a lesson to introduce sixth-grade students to the concept of marine habitats. As a focus activity he presented two 25-gallon aquariums. Aquarium A has a variety of aquatic plants, 10 snails, and 25 unique varieties of small fishes. Aquarium B contains a variety of aquatic plants, three snails, and five small fish of the same variety. To implement the lesson, he organized the students in groups of five. Once the groups were formed, students were asked to discuss the appropriateness of the combination of aquatic plants and number of animals. The ultimate goal is to predict the success of the habitats, including successful breeding among animals. After a lengthy discussion, students agreed that Aquarium B had a better chance to establish and maintain a successful habitat. What information led students to arrive at this conclusion?

Explanation

A healthy aquatic system should have sufficient space and conditions to support life. Aquarium A has a large number of animals for a small 25-gallon aquarium. Overpopulation can pollute the environment and affect the balance of the ecosystem. The size of the aquarium Competency 035

Submit
33. Mike is a four-year-old monolingual, Spanish-speaking child. His parents read Spanish books to him every day. As a result of their effort, Mike has mastered directionality in reading and basic decoding skills. He can also decode words by linking two syllables. Based on his literacy growth, what might be the next logical step for the parents to promote his growth?

Explanation

Reading to children in any language can promote interest in reading and decoding skills. Promoting reading comprehension is the next logical step. Once the child masters these basic skills, these can easily transfer to the second language. (A) is incorrect because reading to children should not be interrupted, especially during the preschool years. Reading to children is beneficial even for advanced readers. (C) is incorrect because the scenario does not provide evidence to suggest that the child is ready for more challenging first-grade books. (D) is incorrect because reading skills acquired in the first language can transfer to the second language. Additionally, students should be exposed to reading in the dominant language first. Competency 007

Submit
34. In a group of people, there are 18 blondes, 17 brunettes, and 5 red heads. What is the probability of selecting a red head from the group?

Explanation

There are a total of 40 individuals in the group. Of the 40 people in the group, 5 are red heads. This gives a ratio of 5/40, which may be simplified to 1/8. Competency 015

Submit
35. The Mayflower Compact was one of the earliest agreements to

Explanation

The Mayflower Compact was drawn up and signed by the Pilgrims aboard the Mayflower. They pledged to consult one another to make decisions and to act by the will of the majority. It is one of the earliest agreements to establish a political body and to give that political body the power to act for the good of the colony. Competency 023

Submit
36. Perform the indicated operation: (–36) – 11.

Explanation

When subtraction involves any negative numbers, a good rule to use is, "Don't subtract the second number. Instead, add its opposite." Using that rule, the original expression, (–36) – 11 becomes (–36) + -11. To be "in debt" by 36, then to be further "in debt" by 11, puts one "in debt" by 47, shown as –47.Competency 013

Submit
37. Which of the following is LEAST likely to lead to illness and disease?

Explanation

The only answer that is not risky behavior for causing illness and disease is (B) keeping the body hydrated by drinking a lot of water. All other answers put the body at risk for illness and/or disease. Competency 044

Submit
38. Which of the following sets of numbers is not an integer followed by its square?

Explanation

An integer number is a whole number that is either positive or negative. Remembering that a negative times a negative gives a positive means that any of the answers could be possible and we cannot rule any out by process of elimination. Knowing that 8 times 8 gives 64 means both (A) and (B) are true. Similarly, –9 times –9 results in 81 (D) meaning 81 is true. However, 6 times 6 gives 36, not 32. This means that (C) is not a true statement and is the solution to the problem. Competency 015

Submit
39. Identify the statement that best describes the country of Iraq.

Explanation

Iraq is a Muslim nation with multiple ethnic groups within its borders. The largest groups are the Arabs, consisting of Shiite and Sunni Muslims. The Kurds are the largest minority group. Competency 021

Submit
40. How is knowledge of a student's instructional reading level useful when planning reading instruction?

Explanation

Instructional reading levels are texts that a student can read with at least 90-94% accuracy. Students should be matched with text that has a readability level that is best suited for their instructional level for supported instruction such as in guided reading. (B) is incorrect because teachers should use grouping for instruction in a more flexible fashion. (C) is incorrect because the student may have differing levels of ability in other subject areas. (D) is incorrect because it does not provide an example of more individualized reading instruction. Additionally, whole class reading does not take into account each student's reading level. Competency 012

Submit
41. Identify the statement that best describes the relationship between the eight social studies strands and the TEKS in the implementation of the Texas curriculum.

Explanation

The social studies strands are systematically integrated throughout the social studies state curriculum in grades K–12. (A) is incorrect based on the previous explanation. (C) and (D) are incorrect because the social studies strands are used in public education as well as in higher education. Competency 019

Submit
42. George placed a metal spoon on an open flame and the spoon became so hot that it burned his hands. What scientific principle represents this type of energy transfer?

Explanation

Conduction is a form of heat transfer. The metal of the spoon serves as a conductor of the heat emanating from the open flame. (A) is too generic because radiation describes different kinds of energy produced naturally by the sun and artificially by microwaves and cellular phones. (C) is incorrect because it does not describe the type of transfer of energy presented in the scenario. However, convection is also a form of heat transfer carried through movement of matter. For example, the heating of water in a pan creates movement of the liquid from the bottom to the top, forcing cold water down to the bottom of the pan, where it is heated. Kinetics (D) is a term used to describe movement, not energy transfer. Competency 033

Submit
43. Mr. Chapman is a third-grade teacher who is working at a school that is implementing a balanced reading program. He will use all of the following reading strategies in his classroom EXCEPT

Explanation

A balanced reading program is one in which the teaching of reading requires solid skill instruction, including several techniques for decoding unknown words. However, having students decode words on their own is not considered to be the cornerstone of such a program. Competency 004

Submit
44. Mr. Obama uses monosyllabic words to present the concept of onset and rimes. Identify the pair of words that best represents this concept.

Explanation

Onsets represent the first phoneme of a syllable or a monosyllabic word like the words presented in choice (D). Rimes follow the onset and are linked to the concept of word families. The rime ank can be used to create multiple words, like blank, tank, rank, and flank. Choices (A) and (C) are generally used to represent verb tenses and cannot be considered the typical rime or word family. Choice (B) is incorrect because the two words do not represent onset and rimes. Instead, the example of very-berry represents a minimal pair—two words that differ in only one phoneme. Competency 003

Submit
45. A rubric to rate the writing skills of children in first grade should not place heavy emphasis on the mastery of spelling because

Explanation

Children go through predictable stages of spelling development. During the initial stages, children invent words and use phonics skills as a foundation for spelling, which often results in nonstandard spelling. (B) and (C) are incorrect because spelling might not be the most important element in writing, but it is definitely important for effective writing. (D) is incorrect because it does not address the question. It just indicates that spelling requires phonics and structural rules, but it does not say why a rubric should not place heavy emphasis on spelling. Competency 010

Submit
46. How many lines of symmetry do all non-square rectangles have?

Explanation

If you can fold a two-dimensional figure so that one side exactly matches or folds onto the other side, the fold line is a line of symmetry. The figure in the problem is a non-square rectangle meaning two of the sides are longer than the other two. Because of this, the shape only has two lines of symmetry. Folding the object from one corner to its opposite corner would not result in a fold where the sides were on top of each other. Competency 016

Submit
47. All of the following are characteristics of emergent readers EXCEPT:

Explanation

Emergent readers will make use of various strategies for understanding print and make meaningful use of texts. They begin to develop the awareness of the story's structures. They will also use illustrations to understand what they are reading and use visual imagery to represent their stories. Although some emergent readers may begin to use self-correction, it is typically not a characteristic of emergent readers. Competency 003

Submit
48. Why might children in prekindergarten and kindergarten have problems understanding how the Earth is represented in globes and flat representations?

Explanation

Maps use symbolic representation, and children at that age rely mostly on concrete experiences for learning. The abstraction typical of maps' legends and other symbolic representations might not be developmentally appropriate for this age group. (B) is a plausible statement but not the best answer. Children initially will be inclined to use the globe as toys and pay attention to colors, but eventually they will understand its function through classroom instruction. (C) is incorrect because it presents an opinion not supported in the scenario. (D) is probably a true statement: Children will have problems conceptualizing how the Earth can be represented in a small 12-inch globe, but this choice is not the best answer because it addresses only one component of the question—globe representation. Competency 019

Submit
49. What is the main function of Earth's atmosphere?

Explanation

The main function of the atmosphere is to serve as a buffer between space and the Earth's crust. This buffer provides the ideal conditions to protect and preserve life on Earth. (B) and (D) present two functions that can be linked to the atmosphere, recycling water and gases; however, they fail to highlight the real function of the atmosphere. (C) is completely incorrect; the atmosphere is above the crust, not beneath it. Competency 038

Submit
50. Identify the statement that best represents the rationale for the implementation of initial reading instruction in the student's native language.

Explanation

Research on second language acquisition suggests that a strong literacy development in L1 can facilitate the acquisition of similar levels of proficiency in L2. Some language specific variables together with literacy and metacognitive strategies can also transfer from L1 to L2. Reading skills are not identical in L1 and L2; (B) does not explain the rationale for implementing instruction in L1. (C) is probably an accurate statement, reading can be confusing for ELLs, but this statement does not address the question. (D) represents an opinion that fails to provide the rationale for the implementation of initial reading instruction in L1. Competency 007

Submit
51. Mr. Rovira guides five-year-old children in the recitation of the following utterance: "Round the rugged rock the ragged rascal ran." Children have fun with this activity and try to say it without errors. What literary technique is he using?

Explanation

Alliteration is a technique to emphasize the connection between the consonant and the sound that it represents. In this particular case, the tongue twister is used to emphasize the sound of the /r/. (B) is incorrect because the technique goes beyond the use of a tongue twister. Tongue twisters are examples of alliteration, and they emphasize pronunciation and fluency. (C) is incorrect because the utterance does not represent an example of traditional nursery rhymes. Nursery rhymes are short poems, stories, or songs written to entertain children. Competency 002

Submit
52. The English colonies were established in three regions—the New England Colonies, the Middle Colonies, and the Southern Colonies. The economy of the Southern Colonies was based on

Explanation

The economy of the Southern Colonies was based on the crops of tobacco, rice, indigo, and cotton. Plantations produced agricultural crops in large scale and exploited workers as well as the environment. (A) is incorrect because it was the economy of the Middle Colonies that was based on farming, shipping, fishing, and trading. (B) is incorrect because it was the economy of the New England colonies that was based on farming and very small industries such as fishing, lumber, and crafts. (C) is incorrect because trading was a part of the Middle Colonies economy. Competency 022

Submit
53. Which answer about your metabolism is FALSE?

Explanation

The only incorrect answer about metabolism is choice (B) fasting speeds up your metabolism. Fasting does not speed up metabolism; it shuts metabolism down in order to conserve energy due to little or no calories getting into the body. (A) is true because your basal metabolism (basal metabolic rate) is constantly burning calories and converting energy. (C) is correct because physical activity always stimulates metabolic function. (D) is correct in that metabolism slowly adjusts to the demands placed on the body over long periods of time. Competency 044

Submit
54. When teaching English to ELLs of Spanish background, Ms. Rico introduced a list of Greek and Latin prefixes common to Spanish and English. She explained the spelling patterns and the meaning of each of the prefixes. She presented examples of words that contain the prefixes, and led students to decode these based on the meaning of the prefixes. What decoding strategy is Ms. Rico using?

Explanation

The analysis of the structure of the words for decoding and comprehension is definitely part of structural analysis. Competency 005

Submit
55. To apply the concept of time zones, students need to have a clear understanding of

Explanation

The Earth is divided into 24 zones based on the meridians of longitude, which are determined using the rotation of the Earth and its exposure to sunlight. This rotation creates day and night, and consequently the concept of time. Competency 021

Submit
56. There are two categories of maps: reference maps and thematic maps. An atlas is an example of

Explanation

Reference maps show the locations of places, and boundaries of countries, states, counties, and towns. Atlases or road maps are examples of reference maps. (B) is incorrect because thematic maps show a particular topic such as population density, distribution of world religions, or physical, social, economic, political, agricultural, or economic features. (C) is incorrect because a physical map is a thematic map that shows the topography of the land including land features and elevations. (D) is incorrect because population maps are thematic maps that are used to show where people live in a particular region. Competency 021

Submit
57. Stacy went to the store to purchase a loaf of bread. She originally thought she had only $9.55 in nickels in her purse. She then discovered she had 10 extra nickels in her pocket. How many nickels did she originally have in her purse?

Explanation

This problem provides unnecessary information to solve the problem. The fact that Stacy discovers 10 extra nickels in her pocket is inconsequential to the solution of the problem. Since a nickel is $0.05 and she had $9.55 in nickels in her purse, divide $9.55 by $0.05 to obtain the number of nickels she had in her purse. Competency 015

Submit
58. A two-year-old child using a crayon to scribble on manila paper is focusing on

Explanation

Two-year-old children are primarily concerned with sensory experiences in their earliest art activities. (A) is incorrect because children do not begin to self-express in their artwork until around age five years. (B) is incorrect because children do not begin to use personal symbols in their artwork until around age four years. (D) is incorrect because children do not begin to repeat themes or be concerned with photographic realism until the middle years of elementary school. Competency 042

Submit
59. A compass rose is a design printed on a chart to

Explanation

A compass rose is a design printed on a chart or map for reference. It shows the orientation of a map on Earth and shows the four cardinal directions (north, south, east, and west). A compass rose may also show in-between directions such as northeast or northwest. (B) is incorrect because a map scale shows the distance between two places in the world. (C) is incorrect because features such as elevations and divisions are represented by different colors. (D) is incorrect because the ratio of the distance between two points on the earth and the distance between the two corresponding points on the map is represented by a scale and not by a compass rose. Competency 021

Submit
60. Which of the following sets of symptoms best describes heat stroke?

Explanation

Heat stroke is best recognized and described by red, hot, dry skin due to lack of hydration-a condition not allowing the body to cool itself efficiently. Unconsciousness is another sign of heat stroke in extreme situations. (A) does not include the typical signs of heat stroke rather heat exhaustion which is not as severe. (C) and (D) may accompany heat stroke but are not by themselves primary indicators of the problem. Competency 044

Submit
61. Running records, teacher observations, and speaking checklists are examples of what type of assessment?

Explanation

All three instruments, running records, teacher observations, and the checklist, are examples of informal assessment. (A) is incorrect because at least one of the instruments, running records, is not a teacher-developed instrument. (C) is incorrect based on the information previously stated. (D) is incorrect because all three instruments require some level of subjectivity. Competency 012

Submit
62. Which of these numbers/number representations are irrational numbers?

Explanation

An irrational number is a number that cannot be expressed as a fraction. Pi is one of the most well-known irrational numbers. Additionally, the square root of 2 and Euler's number (e) are well-known numbers that are irrational (at no known point does a pattern appear in the decimals of these numbers). Competency 018

Submit
63. Vocabulary development is a key predictor of success in reading. Identify a strategy that parents can use with preschool children to promote vocabulary development in a fun and relaxed environment.

Explanation

The use of flash cards with pictures representing concepts and concrete objects can definitely be effective in teaching vocabulary to preschool children. (B) is incorrect because parents can teach a word every day to the child, but the child might not be able to participate effectively because preschoolers generally do not have the vocabulary development to teach parents new words. (C) is also incorrect because preschoolers might not have the necessary vocabulary development to participate in a game dealing with antonyms and synonyms. (D) is incorrect because the practice of memorizing vocabulary is quite boring and ineffective for children of all ages. Competency 005

Submit
64. Alternative assessment, such as verbal reports, laboratory practical exams, story writing, developing advertisements or brochures, constructing a concept map, creating drawings or models, developing a play or skit, can provide a better view of the students' performance in science. Why are these assessment practices effective?

Explanation

Alternative assessment describes the process of gathering performance data as part of instruction. This type of assessment is student-centered and authentic in nature. (B) is incorrect because alternative assessment is generally formative in nature, and it does not have to use instruments approved by the State. (C) is incorrect because it is difficult to determine if all the data collected is easily collected and evaluated. (D) is incorrect because alternative assessment does not emphasize students' weaknesses only. Instead, it tries to assess the overall performance of students in real-life and meaningful situations. Competency 029

Submit
65. Convergent research on linear versus curvilinear rhetorical patterns shows that Spanish-speaking English language learners and young children in general have a tendency to follow a curvilinear approach in writing. What strategies can teachers use to support these students?

Explanation

During the prewriting stage, students should be guided to develop an outline that reflects the order of the ideas in the composition. They also can benefit from guiding questions to keep them focused on the topic and the intended audience. Choice (A) is incorrect because learning about grammar structures and pronunciation will not necessarily affect the organization and coherence of the composition. (C) is incorrect because it addresses speaking ability as opposed to the writing process. (D) is a strong distracter but not the best answer. Modeling effective writing is always a good strategy, but examples only might not provide the necessary guidance to make permanent changes in the students' writing style. (B) is a better choice because it provides a strategy that, once learned, can be used in multiple future situations. Competency 010

Submit
66. After devouring its prey, a crocodile opens its mouth to allow a bird to remove and eat the pieces of meat wedged between the teeth. Once the pieces of meat are removed, the bird flies away. What kind of relationship does this scenario create?

Explanation

Both animals, the crocodile and the bird, benefit from this relationship. The bird gets food and at the same time cleans the teeth of the crocodile. (A) is incorrect because the term commensalism describes a relationship in which only one organism benefit, from the interaction, while the second organism is not affected by the interaction. (B) is incorrect because in a parasitism relationship; only one organism benefits from the interaction. The term predator-prey (C) relationship does not describe the interaction between the bird and the crocodile. Competency 027

Submit
67. The state song of Texas is

Explanation

In 1929, the Texas State Legislature adopted Texas, Our Texas as the state song. (A) is incorrect because the song Home on the Range is the state song of Kansas. (B) is incorrect because the song Deep in the Heart of Texas is a well-known song elaborating on the qualities of the state. (C) is incorrect because the song The Yellow Rose of Texas is a well-known song in the state. It is a patriotic song popularized by Texan soldiers during the American Civil War. Competency 043

Submit
68. The Virginia House of Burgesses was established in Jamestown to

Explanation

The Virginia House of Burgesses was the first legislature established in the English colonies. It was established in 1619 and became the first form of government in the colony. (A) is incorrect because the U.S. government did not exist during this time (1619). (B) is incorrect because England regulated trading with the colonies. (D) is incorrect because, there was no official entity in the colony to regulate the establishment of religions. Competency 023

Submit
69. The technical term used to describe the type of vocabulary needed to understand social studies concepts is known as

Explanation

Cognitive or academic vocabulary is the vocabulary needed to understand the concepts of school. In other words, it is the vocabulary of teaching and learning. Based on this explanation, options (A), (B), and (D) are incorrect. Competency 019

Submit
70. Scenario: Mr. Jetter organized a lesson to introduce sixth-grade students to the concept of marine habitats. As a focus activity he presented two 25-gallon aquariums. Aquarium A has a variety of aquatic plants, 10 snails, and 25 unique varieties of small fishes. Aquarium B contains a variety of aquatic plants, three snails, and five small fish of the same variety. To implement the lesson, he organized the students in groups of five. Once the groups were formed, students were asked to discuss the appropriateness of the combination of aquatic plants and number of animals. The ultimate goal is to predict the success of the habitats, including successful breeding among animals. Students also agreed that Aquarium B has a better chance of successful breeding among the fish. What information did students use to arrive at this conclusion

Explanation

Fish of the same kind have a better chance of reproducing themselves. Aquarium B contains fish of the same kind, while aquarium A has different varieties. Competency 035

Submit
71. The most advanced pre-Columbian civilizations of Mesoamerica were the

Explanation

The Maya and the Aztecs occupied the area of Central America and Southern Mexico called Mesoamerica. Both groups were accomplished builders, astronomers, and mathematicians. (A) is incorrect because the Inca civilization was not a Mesoamerican group. They developed an advanced civilization in South America, in present-day Peru and Ecuador. (C) is incorrect because only the Toltecs were from Mesoamerica; the Pueblo Indians were from present-day New Mexico in North America. (D) is incorrect because only the Olmecs were a Mesoamerican group. The Iroquois civilizations developed in North America. Competency 020

Submit
72. In music, the words "up" and "down" are usually associated with

Explanation

Pitch describes how high (up) and low (down) sounds are produced. (A) is incorrect because it describes characteristics of rhythm. (B) is incorrect because it describes characteristics of dynamics. (D) is incorrect because it describes characteristics of weight. Competency 043

Submit
73. What song has the same rhythm as Happy Birthday?

Explanation

The rhythm is typically called the melodic rhythm or word rhythm found in the song. The beginning of Happy Birthday ("Happy birthday to you") has the same word rhythm as "Oh, say can you see." (A) is incorrect because the word rhythm "My country 'tis of thee" is not the same as "Happy Birthday to you." (C) is incorrect because the word rhythm "Take me out to the ball game" is not the same as "Happy Birthday to you." (D) is also incorrect because "Oh beautiful for spacious skies" is not the same as "Happy Birthday to you." Competency 043

Submit
74. A card is drawn from a deck of cards, what is the probability that the card is a queen or a black four?

Explanation

In a normal deck of cards there are 52 total cards. Of the 52 cards, there are four suits and 13 cards per suit. Therefore, there are four queens in the deck. Additionally, of the four suits, two are black and two are red. So, there are two black fours giving a grand total of six cards of 52 that meet the criteria. Competency 017

Submit
75. If in the United States we are enjoying a warm winter, what season are the people of Brazil having?

Explanation

Brazil is located in the Southern Hemisphere, and the United States is located in the Northern Hemisphere; thus, if the United States is in the winter season, then in Brazil it is summer. (A), (C), and (D) are incorrect based on the previous explanation. Competency 021

Submit
76. Germaine, a fourth-grade student, notices that the shape of South America is similar to the shape of Africa. To demonstrate this, he cut out the South American map, placed it next to the map of Africa, and found an almost perfect match. He was amazed by his discovery and asked the teacher if Africa was once part of South America. What type of theory or scientific principle can the teacher introduce to make Germaine's discovery a teachable moment?

Explanation

The teacher can discuss the theory of continental drift, which gave rise to the current theory of plate tectonics. According to this theory, the crust of the Earth is broken down into several floating tectonic plates, which move and change locations. (A) is incorrect because it describes how organisms evolve throughout history. It also alludes to the work of Charles Darwin. (B) is incorrect because it refers to the religious belief that the universe was created by a supreme being. (C) describes the process by which organisms living on one continent manage to migrate to other continents or to other regions within a continent. Competency 038

Submit
77. Vinegar is composed of two substances—acetic acid (a colorless liquid) and water. This combination of substances is an example of a

Explanation

A mixture is a physical combination of two or more substances that retain their own chemical properties; i.e., the mixture can be separated into the original substances—acetic acid and water. (A) is incorrect because in a compound the substances are chemically combined, and in the case of vinegar the substances are only mixed physically. (C) and (D) are incorrect because they lack the specificity required of the answer. Competency 021

Submit
78. In her first-grade class, Ms. Swanson wants to present the fact that scientists can be male or female, and can come from all ethnic and racial groups, and countries in the world. What might be the best strategy to accomplish that goal?

Explanation

Reading and discussing the biographies and the accomplishments of scientists from a variety of backgrounds can minimize the stereotypical views that children may have about them. (A) and (D) are incorrect because both address the gender misconception only. Providing generic information about the diversity in the science field (B) might not be the best strategy for first graders. They might need more tangible evidence of the information. Competency 025

Submit
79. If a can weighs 14 oz., how many cans would you need to have a ton? (Round your answer to the nearest ones place and pick the best answer.)

Explanation

An easy way to solve this problem is to use basic algebra. Knowing that there are 16 oz. in a pound and that there are 2,000 lbs. in a ton helps ease the difficulty of the problem. We want to find out the number of cans x it will take to obtain a ton. Therefore, we have . If both sides of the equation are multiplied by 16 and then we divide both sides by 14, we will obtain the approximate number of cans it will take to obtain one ton. . We see that many of the answers are close to this value. When we round this number, we will obtain 2,286. Competency 015

Submit
80. Which of the following is a locomotor skill?

Explanation

Locomotor activities describe the types of movement that children use to move from one place to the other. In this case, hopping is the only choice that accomplishes this goal. (A) is incorrect because bending is not a locomotor activity, rather a non-locomotor activity. (B) and (C) are also incorrect because they are manipulative skills requiring interaction with equipment or objects. Competency 045

Submit
81. Xylophones, metallophones, and glockenspiels are instruments associated with what classroom musical approach?

Explanation

The Orff approach uses both unpitched rhythm instruments (e.g., wood blocks, triangles, etc.) and melodic or barred instruments (e.g., xylophones, metallophones, and glockenspiels). The Kodály Method (B) is incorrect because the primary goal is to teach music literacy and singing is the vehicle to achieve this goal. Mariachi (C) is a group of musicians that play violins, trumpets, a Spanish guitar, and a guitarrón. Tejano music (D) features the accordion, and the Tejano orchestra has been influenced by Mexican, Cuban, German, and Czech brass bands. Competency 043

Submit
82. Identify the informal activities/instruments used to assess oral communication skills in Pre-K students.

Explanation

Checklists, retelling stories, and the use of anecdotal records are examples of informal assessment measures commonly used to assess the development of communication skills among ELLs. The informal nature of these assessment strategies eliminates the stress associated with formal assessment procedures. Multiple-choice, cloze tests, and informal reading inventory (B) are not generally used to assess oral communication skills. Audiotaping can be used to assess oral language development, but written cloze tests and standardized achievement tests (C) are not. Repetition drills, choral reading, and chants (D) are generally used to assess pronunciation, which is only one component of oral communication skills. Competency 005

Submit
83. Mrs. Cameron plans to buy carpeting for her living room floor. The room is a rectangle measuring 14 feet by 20 feet. She wants no carpet seams on her floor even if that means that some carpeting will go to waste. The carpeting she wants comes in 16-foot-wide rolls. What is the minimum amount of carpeting that will have to be wasted if Mrs. Cameron insists upon her no-seams requirement?

Explanation

Since Mrs. Cameron does not want any seams in her carpet, the carpet must be 20 ft long (at least) to cover the entire space. Since the room is only 14 ft. wide and the carpet is 16 ft wide, there will be 2 ft of wasted carpet for the entire length of the room (20 ft). Therefore, the amount of carpet that is wasted is 20 x 2 = 40 ft.2 of carpet. Competency 016

Submit
84. In what grade does the state curriculum officially introduce children to Texas history?

Explanation

TEKS introduces social studies beginning with what the child knows to what the child does not know. Students begin studying about themselves and from there they study the school, community, and, finally, the state. The focus on TEKS for fourth grade is the history of Texas. (A) is incorrect because TEKS is available only in K–12. For prekindergarten, teachers use instructional guidelines. (B) is incorrect because the focus of first grade is the school and the community. (C) is incorrect because in grade 3 the focus is the individual and how each affects the community. Competency 019

Submit
85. Jamie rolls a pair of dice hoping to get an odd number. What is the probability that the sum of the dice will show an odd number?

Explanation

Knowing that with a pair of dice there are 36 possible outcomes allows us to view all of the possible outcomes. Of the possible outcomes, half are odd and half are even. Competency 017

Submit
86. The Sumerians, Akkadians, Babylonians, and Assyrians were some of the civilizations that flourished in Mesopotamia—the land between the rivers. What rivers are being alluded to in this statement?

Explanation

The convergence of the Tigris and the Euphrates created a fertile region where some of the greatest civilizations of the world emerged. This part of the world has been called the Fertile Crescent, and it is part of the area called the Cradle of Civilizations. (A) is incorrect because the Nile gave birth to the Egyptian civilization, and the Amazon River is located in South America, far away from Mesopotamia. (C) is incorrect because the Indus River is located in modern-day India. (D) is incorrect because the Volga and Danube rivers are located in Europe. Competency 020

Submit
87. In the early grades, mathematic and spatial reasoning are introduced in learning centers through the use of manipulatives. What is the main advantage of using learning centers to introduce mathematics and spatial reasoning?

Explanation

Learning centers allow children opportunities to explore mathematics reasoning in a play and low anxiety format. (B) is incorrect because it addresses only one component of the benefit—the use of manipulatives. (C) is incorrect because, traditionally, learning centers are not used to teach deductive reasoning. Instead, students learn inductively through the use of manipulatives and the input from teachers. (D) is incorrect because learning centers do not use a highly structured format. On the contrary, learning centers are designed to be a low anxiety environment in which children can explore on their own. Competency 013

Submit
88. There are 16 more apples than oranges in a basket of 62 apples and oranges. How many oranges are in the basket?

Explanation

This problem is easily solved by using some basic algebraic reasoning. Since we are interested in determining the number of oranges that are in the basket, we will set this as a variable called o. The number of apples, a, and the number of oranges, o, sum to a total of 62. We also know that there are 16 more apples than oranges (a = o + 16). This gives o + (o + 16) = 2o + 16 = 62. Solving for o yields 23 oranges in the basket. Competency 015

Submit
89. Two coins are tossed at the same time. What is the probability that only one head is obtained in each of the tosses?

Explanation

Since tossing two separate coins does not effect the outcome of the other coin, we know that the probability of getting a head on either coin is 1/2. The probability of the outcome is then 2/4 or 0.5. Competency 017

Submit
90. The Declaration of Independence consists of a preamble or introduction followed by three main parts. The first part stresses natural unalienable rights and liberties that belong to all people from birth. The second part consists of a list of specific grievances and injustices committed by Britain. What does the third part announce?

Explanation

The Declaration of Independence pronounced the colonies free and independent states. It consists of a preamble or introduction followed by three main parts. The third part announces the creation of the new country. (B), (C), and (D) are incorrect because these choices stress rights that are not explicitly stated in the Declaration of Independence. The unalienable rights, stated in the first part of the Declaration of Independence, are life, liberty, and the pursuit of happiness. Competency 023

Submit
91. The state curriculum for fine arts and for visual arts is organized around the following four strands:

Explanation

Perception, creative expression, historical/cultural heritage, and critical evaluation are precisely the four strands identified in the state curriculum for fine arts and visual arts. (A) is incorrect because, although art production, appreciation, and evaluation may be included in the four strands under slightly different terminology, art advocacy is not. (B) is incorrect because, although sensory awareness, creativity, and art production may be included in the four strands under slightly different terminology, crafts are not. (D) is incorrect because the wording of each strand has been changed. Competency 042

Submit
92. A globe is a scale model of the Earth shaped like a sphere. A globe shows sizes and shapes more accurately than

Explanation

A globe is a scale model of the Earth shaped like a sphere. Because a globe is the same shape as the Earth, it shows sizes and shapes more accurately than a Mercator projection map (a flat representation of the Earth). (A) is incorrect because a compass rose is a design used to show orientation and not a representation of the Earth. (C) is incorrect because a map scale is used to show the distance between two places in the world. (D) is incorrect because thematic maps are not used to show the size and shape of the Earth. Competency 021

Submit
93. Identify the main benefit of the shared book experience.

Explanation

The overall purpose of the shared book experience is to guide children to be successful in reading and to develop an interest in learning to read. The story is read to the students in a very supportive environment using visuals to enhance comprehension. The main goal is to make reading an enjoyable activity and to motivate children to read on their own. (B) is incorrect because reading with the whole class is a by-product of the shared book experience, but it is not the main goal. (C) and (D) are true statements. Traditionally, books used in shared reading use big letters and attractive pictures, and children like big books, but these features are not the main reason for conducting shared reading. Competency 004

Submit
94. Why are Indian tribes from the Central and Great Plains of Texas better known than their counterparts from the Coastal Plains?

Explanation

The Apache and the Comanche domesticated the horse and became skillful hunters and warriors. These skills allowed them to fight the whites for many years for control of the Central and Great Plains of Texas. (A) is incorrect because the tribes from the plains were nomads and did not develop an advanced civilization. (B) is incorrect because both the Comanche and the Apache were nomads and did not leave permanent constructions. (D) is incorrect because the tribes from the area had to hunt for survival and they had to move continuously to find adequate food supplies. Competency 020

Submit
95. Vocabulary development is a key predictor of success in reading in the content areas for all students, but especially for children new to the English language. Identify the best strategy to introduce vocabulary development for ELLs from diverse cultural and linguistic backgrounds.

Explanation

The best way to teach vocabulary to ELLs is to present the word together with a visual or concrete representation—a word concept. The introduction of word concepts can avoid any possible cultural conflict with similar concepts in the students' cultures. (B) is incorrect because translation can teach the word but fail to teach the concept that the word represents. (C) is incorrect because it would be very difficult to teach every single word in a contrasting fashion. Plus, most ESL classes contain speakers of various languages, which makes this activity impractical. (D) is incorrect because with ELLs, we cannot deemphasize the role of vocabulary development. Using context alone to obtain meaning will not work for children new to the English language. Competency 008

Submit
96. Identify the most appropriate strategy(ies) to meet the needs of children at the emerging stage of writing development.

Explanation

Reading a story and asking a child to retell it promotes interest in writing. When the teacher writes down the dictated story, the child can see the connection of oral and written work. (A) and (B) are incorrect because the children at the emerging stage don't have sufficient command of written language to correct their own writing or write a composition based on prompts. (D) is incorrect because reading for 30 minutes a day without any kind of explicit or implicit writing support might not be effective for children at the emerging stage for writing. Competency 009

Submit
97. How can teachers minimize the danger of working with chemicals in their pure form?

Explanation

The use of household items in place of pure chemicals can reduce the danger for children. Moreover, its use can make science more relevant and practical for students. Avoiding experiments (A) with chemicals (D) is not an option in the upper grades. True scientific experimentation often requires the use of chemicals, thus, substituting the required chemical with plain water is not an option either (C). Competency 024

Submit
98. As part of the celebration of Martin Luther King Day, fifth graders are getting ready to implement readers' theater on a book about the Civil Rights Movement. In preparation for the presentation, the teacher guides the students through several activities. Which of the preparation activities is less likely to help them get ready to implement readers' theater?

Explanation

Readers' theater is a student-centered activity. They read the story, summarize it, work in groups, develop the characters and story line, and implement the activity. While watching a video version of the story (D) can help students get into the characters of the story, this activity is not required to implement readers' theater. Thus, options (A), (B) and (C) are typically used as part of the preparation to implement reader's theater. Competency 006

Submit
99. Two paper bags are each filled with four blue marbles and four red marbles. What is the probability of selecting a blue marble from the first bag and a blue marble from the second bag?

Explanation

There are equal numbers of blue and red marbles in each bag. For a single bag on a single draw, the probability of pulling a red or a blue marble is equal to 4/8 or 1/2 . Selecting a marble from one bag does not affect the probability of selecting a specific marble from the other bag, therefore these events are said to be independent. When dealing with the probability of independent events, one may multiply the probabilities together to obtain the overall probability. In this case, we know the probability of selecting a blue marble is 1/2 for the first bag and it is also 1/2 for the second bag. Therefore, the overall probability of the independent events is 1/4. Competency 017

Submit
100. An example of a prime number is

Explanation

A prime number is a number whose only factors are one and itself. Even numbers greater than 2 can always be factored by 2, eliminating (B). (A) can be factored as 3 times 3 and (C) can be factored as 7 times 7. Therefore, (D) must be the right answer as it only has factors of 1 and 67. Competency 014

Submit
101. In which grade should pupils' experiences in critically evaluating their own art and the art of others begin?

Explanation

The state curriculum states that experiences in critical evaluation for kindergartners should include expressing ideas about personal artworks, and the artworks of peers or professional artists. Critical evaluation is one of the four strands in the state curriculum for fine arts and visual arts. (B) is incorrect because the standards indicate that experiences in critical evaluation should begin in kindergarten rather than first grade. (C) is incorrect because the standards indicate that experiences in critical evaluation should begin in kindergarten rather than second grade. (D) is incorrect because the standards indicate that experiences in critical evaluation should begin in kindergarten rather than third grade. Competency 042

Submit
102. Identify the statement that best describes the connection between reading and writing.

Explanation

All language skills are interrelated, including reading and writing. Given appropriate instruction, the skills of reading and writing can be introduced and acquired concurrently. Current research does not support the idea that children learn language skills in a sequential manner (A). Based on the explanation given, options (C) and (D) are incorrect. Competency 005

Submit
103. The main purpose of an interactive/dialogue journal is to provide children with opportunities to

Explanation

The use of interactive journals allows children opportunities to communicate in meaningful and real-life situations. (A) and (D) are incorrect because listening and speaking abilities are not emphasized in interactive journals. (C) is incorrect because corrective feedback is not encouraged in communication activities. Teachers can provide indirect corrective feedback through modeling. Competency 010

Submit
104. Scenario: Corals are invertebrates that generally live in a symbiotic relationship with algae. Some of them build reefs, others do not. Most of them filter water to obtain nutrients for survival. The term symbiotic in this case means that corals

Explanation

Corals live in a symbiotic relationship with algae because both exchange nutrients or other kinds of services in order for both to survive. (A) is incorrect because corals are not parasites living off algae. Instead, parasites and corals provide support services to each other for their mutual survival. (B) and (D) might be true statements, but they do not define the term symbiosis. Competency 036

Submit
105. Newly fluent readers can read with relative fluency and comprehension. Which cuing system is NOT one they would use to obtain meaning from print?

Explanation

Semantic, structural, and visual cuing systems are all used by newly fluent readers to aid in their comprehension of texts. They also make use of graphophonemic cuing systems. These are used to self-monitor what they are reading as well as to begin to correct their errors without much support. Competency 003

Submit
106. Identify the strategies that lead the child from the stage of learning to read to the stage of reading to learn.

Explanation

The main purpose of reading to learn is to obtain content information efficiently and effectively. One way to accomplish this task is to make students aware of the format used in the content areas and to guide them to retrieve the information by reading for the main idea or scanning for information. Choice (A) is incorrect because it addresses only one component of the process—vocabulary development. (C) is incorrect because it does not address the issue of the complexity of expository writing. (D) is incorrect because it addresses only the issue of vocabulary development—connotation and denotation. Competency 008

Submit
107. In a science class for sixth graders, Ms. Anderson posted a list of controversial scientific issues including global warming, searching for oil in natural reserves, and genetically engineered food. She asked students to take a position or a stand on one of the issues, and to come to class ready to defend their position. What is the educational benefit of this kind of assignment?

Explanation

Learning about computers and search engines (A) is a derivative of the process; it does not constitute the main benefit of the activity. (C) and (D) are incorrect because they represent a generic value and byproducts of the activity. Competency 026

Submit
108. Introducing the multiple versions of stories like Cinderella can help students understand how a theme can be developed from different points of views. Moreover, this kind of literature can help students in the development of

Explanation

Comparing and contrasting variations of a fairly tale, like Cinderella, can foster higher order thinking skills and critical reading. Students will have to analyze the new story to determine how it related to the traditional version, and how the theme is treated. (B), (C), and (D) are not focusing specifically on such higher-level reading as it relates to comparing and contrasting across multiple texts. Competency 007

Submit
109. Simplify: 6  * 2  +  3  ¸  3

Explanation

The order of operations must be obeyed here. Remembering the saying Please Excuse My Dear Aunt Sally (PEMDAS) allows us to remember the order in which mathematical operations must be carried out, Parentheses Exponent Multiply Divide Add Subtract. Following this one will multiply 6 by 2 to obtain 12. Then, one will divide 3 by 3 obtaining 1. Finally, one will add the two results together to obtain 12 + 1 = 13. Competency 013

Submit
110. Which of the following words can be used as a good example for homonyms?

Explanation

Homonyms are words that have the same spelling and pronunciation, but have different meanings. The only pair of words that is a true homonym is (B). In this case, the word club can refer to a night club or a wooden stick. Option (A) and (C) are incorrect because they represent two different words. Option (D) represents an example of homophones, words that are pronounced in the same way, have different meanings, and are spelled differently. Competency 002

Submit
111. The use of hot plates and burners are initially allowed in

Explanation

The Texas Essential Knowledge and Skills (TEKS) allows the use of burners and hot plates beginning in fifth grade. Based on this information, the rest of the options are incorrect. Competency 025

Submit
112. The Fifteenth Amendment of the U.S. Constitution was ratified in 1870 to grant

Explanation

The Fifteenth Amendment granted black males the right to vote. (A) is incorrect because the voting right was given to black males only, not women. (B) is incorrect because the Fourteenth Amendment granted citizenship to former slaves. (C) is incorrect because the Thirteenth Amendment granted freedom to slaves. Competency 023

Submit
113. Identify the number of syllables and the number of phonemes present in the word thought.

Explanation

The word thought is a long word, even though it is monosyllabic. It has three phonemes. The word contains two consonant diagraphs, th and ght, and a vowel digraph, ou, representing one sound each for a total of three sounds. Competency 001

Submit
114. Identify the modern-day country that encompasses most of the territory of ancient Mesopotamia.

Explanation

The country of Iraq is located in the region known as Mesopotamia. (A) is incorrect because the country of Iran is located east of the Tigris River. (B) is incorrect because Pakistan is located further southeast of these rivers. (C) is also incorrect because the State of Kuwait is located southwest of the area and south of Iraq. Competency 020

Submit
115. Which is not a critical step to follow when solving a problem?

Explanation

In order to solve a problem we must understand the problem, choose a strategy and/or make a plan, carry out the plan, and check our answer. The only answer that is not a critical component of the process is (D). Competency 018

Submit
116. Scenario: Mr. Mackey has seven ELLs in his class who are having difficulties pronouncing English words containing the letters sh and ch. To support these students, he organizes small-group instruction to address their specific needs. He introduces pronunciation and vocabulary concepts through a chart. The chart is color-coded to guide students in the pronunciation of phonemes; i.e., he uses blue for the /sh/, green for /ch/, brown for /k/. Every week, he adds new words to the chart until he is satisfied that students have mastered the grapheme-phoneme correspondence. The chart for this week is shown below. What might be the advantages of using the chart to teach pronunciation?

Explanation

The chart provides examples of words that follow grapheme-phoneme correspondence. It also provides examples of words that use different graphemes to represent the sounds. This chart provides students with tangible information to help them deal with graphophonemic inconsistencies. (A) is incorrect because this particular chart places more emphasis on graphophonemic inconsistencies than on consistencies. (C) is incorrect because the chart presents words in isolation; thus, it cannot present information about the intonation pattern used in sentences or larger units. (D) is a plausible answer, but it does not capture the true intent of the activity. The chart is designed to deal with specific, inconsistent sounds only. Competency 001

Submit
117. Simplify to a single term in scientific notation: (2 · 10)3 · (6 · 104).

Explanation

To solve this problem, one again must use PEMDAS first on the exponential term. Once this is done, one obtains (2 · 10)3 = 11 · 103(2 · 10)3 = 11 · 103. Then, multiplying the two single-digit numbers (8 and 6), one gets 48. Continuing with the solution, we now can multiply 103 with 104 to obtain 107 by adding the exponents (this can be done because each number has a common base, 10). We are now left with 48 · 107. However, the last step is to write the answer in scientific notation, which is 4.8 · 108. Competency 013

Submit
118. The United States is probably the only industrialized country in the world that uses the English system of measurement in daily life. What is the rationale for using the metric system in scientific research and engineering?

Explanation

The metric system is a very precise system that allows for the measurement of very small amounts of matter. For example, the metric system uses measurements divided into thousands (millimeters or milligrams), while the English system uses larger units of measurement, like inches and ounces. (A) and (B) contain possible true statements and opinions but they fail to address the question. (D) is incorrect; the English system does not have a simple way to compute measures. On the other hand, the metric system is a base 10 system and the computations are typically easier. Competency 016

Submit
119. Scenario: Mr. Mackey has seven ELLs in his class who are having difficulties pronouncing English words containing the letters sh and ch. To support these students, he organizes small-group instruction to address their specific needs. He introduces pronunciation and vocabulary concepts through a chart. The chart is color-coded to guide students in the pronunciation of phonemes; i.e., he uses blue for the /sh/, green for /ch/, brown for /k/. Every week, he adds new words to the chart until he is satisfied that students have mastered the grapheme-phoneme correspondence. The chart for this week is shown below. Based on the information provided in the chart, what is the primary reason for the confusion that ELLs experience with these two phonemes?

Explanation

A visual analysis of columns 4, 5, and 6 shows that the phoneme /sh/ can be represented by at least four different graphemes—ch, s, ss, and t—in words like chef, sure, mission, and caution. The grapheme ch is also inconsistent. Column 3 shows that the grapheme ch represents the sound /k/. This inconsistency affects the ability of ELLs to separate the two phonemes. (A), (B), and (C) are incorrect because the scenario does not provide evidence to suggest that ELLs cannot establish a difference or pronounce the two phonemes, or that they might not be interested in learning the difference between the two. Competency 001

Submit
120. Which of the following were the first mathematicians to impact the development of modern-day mathematics?

Explanation

While other groups made significant contributions to mathematics, the Egyptians and Babylonians (third millennium BCE) were the first groups to make an impact on the development of modern-day mathematics. Competency 018

Submit
121. Learning about computers and search engines (A) is a derivative of the process; it does not constitute the main benefit of the activity. (C) and (D) are incorrect because they represent a generic value and byproducts of the activity. Competency 026

Explanation

The teaching of the writing trait of "voice" can be taught through sharing examples, such as through reading aloud stories by authors who portray and exemplify strong voice in their unique style of writing. Teachers can ask their students what characteristics best exemplify this author's voice across multiple texts. Option (A) focuses less on voice and more on the ways in which the writing is structured. (C) and (D) focus more on syntax, grammar, and punctuation and less on voice, although they are related features to how authors construct voice. Competency 010

Submit
122. Which statement best describes an example of the Language Experience Approach?

Explanation

In the Language Experience Approach (LEA), the teacher is dictating the ideas and thoughts of the students. As the students share ideas, the teacher writes them down, thus connecting speech to print and also modeling how oral language is related to writing. Answer (B) is incorrect because in this approach, it is the teacher who "has the pen" and does the modeled writing as students contribute ideas. Answer (C) is incorrect because the focus is not on oral reading primarily; the emphasis is on both reading and writing for beginning readers with LEA. Answer (D) is incorrect because it focuses on the teacher's own ideas, whereas LEA focuses on the teacher dictating the ideas of the student or students. Competency 009

Submit
123. Characteristics of emergent writers include the following EXCEPT

Explanation

Students who are becoming literate are not yet writing conventionally. However, answers (A), (B), and (C) all represent features and characteristics of emergent writers. As young writers grasp the concept of the alphabetic principle and are able to better map speech onto print in conventional ways, their spelling will become closer approximations to conventional spelling. Competency 009

Submit
124. In 2008 the gross state product of Texas was $1.245 trillion-the second highest in the nation. If Texas were an independent nation, its economy will rank ___________ in the world.

Explanation

In 2008, The Texas economy ranked 12th in the world. The gross state product competes favorably with the gross national product (GNP) of countries like Mexico, India, Australia, and South Korea. Based on this information, (A), (C), and (D) are eliminated. Competency 022

Submit
125. Marcus can separate a word into its individual phonemes and put it back together to recreate the original word. However, he has problems separating words into syllables and identifying the syllable with the primary stress. Based on this scenario, Marcus needs additional support with

Explanation

The child is having problems with word stress and syllabication, and both terms are part of phonological awareness. (B) is incorrect because the child is not having problems with phonemic awareness. He has mastered the ability to separate phonemes and blend them back to recreate the word. (C) and (D) are incorrect because neither of them individually can explain the needs of the child. The reality is that the child needs support in both components; thus, (C) or (D) individually cannot be the correct answer. Competency 002

Submit
126. What is the main purpose of the Federal Reserve System?

Explanation

The main purpose of the Federal Reserve System (FRS) is to keep the banking industry strong enough to ensure a supply of currency. When the banking industry is strong and there is an adequate supply of currency, fiscal stability and economic growth are more likely to occur. However, the direct role of the FRS is not to provide stimulus money to balance the economy (A), promote economic growth (B), nor create inflation and deflation (C) in the country. Policies adopted by the FRS can lead to a healthy economy—or a weak economy resulting in inflation or deflation. Competency 022

Submit
127. Scenario: Corals are invertebrates that generally live in a symbiotic relationship with algae. Some of them build reefs, others do not. Most of them filter water to obtain nutrients for survival. Based on this description, corals belong to which of the following kingdoms?

Explanation

Corals are invertebrates that fall under the animal kingdom. These small organisms can reproduce themselves sexually or asexually. (A) is incorrect because unlike corals, the kingdom Monera is made up of unicellular organisms with very rudimentary cellular organization. (C) and (D) are incorrect because corals are described in the question as invertebrates, a classification used exclusively for members of the animal kingdom. Competency 036

Submit
128. Identify two key historical concepts or characters of the Middle Ages.

Explanation

The feudal system characterized life in the Middle Ages. The attacks of the barbaric tribes forced people to live under the protection of a lord, usually within the walls of a castle. Muslim Turks took Jerusalem in 1095, and a year later, Christians began the Crusades to rescue the Holy Land. (A) is incorrect because both the Greek and Roman empires are part of the Ancient World. (C) is incorrect because one of the events is part of the Ancient World—Stonehenge. (D) is incorrect because Alexander the Great ruled the Ancient World. Competency 020

Submit
129. Use the following figure to answer the question that follows. Assume that point C is the center of the circle. Angles xyz and xCz intercept minor arc xz. The measure of angle xyz is 40°.  

Explanation

Angle xyz is an inscribed angle (its vertex is on the circle). Angle xCz is a central angle (its vertex is at the circle's center). When two such angles intercept (or "cut off") the same arc of the circle, a specific size relationship exists between the two angles. The measure of the central angle will always be double the measure of the inscribed angle. In this case, that means that the measure of angle xCz must be 80°. Thus, minor arc xz also measures 80°. Every circle (considered as an arc) measures 360°. This means major arc xyz measures 280° (360 – 80). The explanation shows that (A), (C) and (D) are incorrect. Competency 016

Submit
130. Living things are classified into five groups: monera, protista, fungi, plants, and animals. Some of these groups are unicellular, while the others are made up of multiple cells; however, there is only one group that has prokaryotic cells. What kingdom contains prokaryotic cells?

Explanation

The kingdom Monera is composed of unicellular organisms and primitive cells called prokaryotes. (B), (C), and (D) are incorrect because all three have more sophisticated cellular systems called eukaryotes. Competency 034

Submit
131. What percentage of a daily diet should be composed of carbohydrates?

Explanation

Complex carbohydrates should comprise at least half of the calories consumed for the healthy diet of an active person (a little less for an inactive person). Carbohydrates are the primary and most efficient source of energy for the body. Based on this, choices (A), (B), and (D) are incorrect. Competency 044

Submit
132. Which of the following is an example of formative evaluation?

Explanation

Formative evaluations are the ongoing measures of how students are doing during regular classroom instruction. They are typically given throughout the school year. Summative evaluations are given at the end of a grading cycle or at the end of the school year. Therefore, options (A), (B), and (D) are incorrect because they represent examples of summative assessment rather than formative (ongoing) assessment.Competency 012

Submit
133. Which of the following is NOT a key element of inquiry-based science learning?

Explanation

Inquiry-based science learning emphasizes active exploration and discovery, not rote memorization. Students are encouraged to ask questions, conduct investigations, and construct their own explanations based on evidence.

Submit
134. Which of the following is NOT a developmentally appropriate practice for promoting literacy in young children?

Explanation

Developmentally appropriate practices emphasize active learning and engagement, allowing children to explore and construct their own understanding of literacy concepts. Worksheets and drills can be tedious and may not be effective for all learners. The other options are more engaging and developmentally appropriate:

Providing a print-rich environment exposes children to the uses and functions of print in their everyday lives.

Interactive read-alouds and discussions promote language development, comprehension skills, and a love of reading.

Encouraging children to express themselves through drawing and storytelling fosters creativity and helps them connect oral language with written expression.

Submit
135. Which of the following is equivalent to 17(64 + 82) – 43?

Explanation

In this problem it is necessary to perform the order of operations as well as look at equivalent representations of numbers. Since there are no exponents in any of the solutions, it may be beneficial to carry this out within the problem before proceeding further. Doing so results in 17(64 + 64) – 64. This may also be written as 17 x 64 x 2 – 64. Since 64 appears in the first multiplication sequence as well as being a subtrahend, it may be factored to produce 64(17 x 2 – 1) = 64(34 – 1) = 64 x 33 . Competency 014

Submit
136. In which grade are the principles of art introduced?

Explanation

The standards state that second graders should use art principles, like emphasis, patterns, and rhythm to create artworks. The principles of art are included in the curriculum strand of Perceptions. (A) is incorrect because the standards indicate that children's first experiences with art principles should occur in second grade rather than kindergarten. (C) is incorrect because the standards indicate that children's first experiences with the art principles should occur in second grade rather than fourth grade. (D) is incorrect because the standards indicate that children's first experiences with the art principles should occur in second grade rather than sixth grade. Competency 042

Submit
137. Students who develop an understanding of the key components of a story (i.e., the setting, characters, and the resolution) have mastered understanding through the use of

Explanation

Story grammar focuses on the key elements in a story. By being able to recollect these key features common to many narrative stories, students can develop their understanding of commonalities across texts as well as their understanding of the specific story they are reading. (A) is incorrect because it focuses mainly on meanings at the sentence or word level rather than across the entire text. (B) is incorrect because it is a specific cognitive strategy the learner uses while reading any text. Shared reading (D) is a way to read a text together and is not specific to analyzing features of a text. Competency 007

Submit
138. Valerie has a bathtub that she needs to fill with water. To fill the tub, she first needs to fill a bucket with water and then dump this into the bathtub. If the bucket is a cylinder with a radius of 6 in and a height of 12 in, how many buckets will it take to fill the 5 ft x 3 ft x 3 ft tub? Pick the best answer.

Explanation

This problem requires critical thinking and some basic knowledge of math. Solving for the volume of the bathtub one obtains 45 ft.3 and the volume of the bucket is p . 0.52 . 1 » 0.7854 ft.3 To find how many buckets of water it will take to fill the tub, we must divide 45 by 0.7854. This gives approximately 57 buckets. Competency 015

Submit
139. During the "engage" component of the 5-E model, students begin the inquiry process. They are guided to engage in scientific inquiry and discover the concepts on their own through hands-on investigation, observation, and data collection. When teachers guide students to discover the scientific principles on their own, what teaching method are they using?

Explanation

In the inductive method, teachers guide students to discover principles in an indirect fashion. In this indirect way of teaching, teachers create the opportunities for the students to develop their own learning. (B) is incorrect because it describes a teaching style in which concepts are presented through direct instruction. The term "hands-on" (C) and "student-centered method" (D) is part of the "engage" stage of the 5-E model, but they do not fully describe the type of strategy being used. Competency 025

Submit
140. Charts, tables, and graphs are commonly used in language arts to

Explanation

Charts, tables, and graphs can be used to present, summarize, and/or complement the message being conveyed. These are not designed to provide detailed explanations (C) of topics covered in class. Regardless of their use, students must be aware that their use should never be intended to entertain (B) or contradict (D) any of their information. Competency 011

Submit
View My Results

Quiz Review Timeline (Updated): Dec 16, 2024 +

Our quizzes are rigorously reviewed, monitored and continuously updated by our expert board to maintain accuracy, relevance, and timeliness.

  • Current Version
  • Dec 16, 2024
    Quiz Edited by
    ProProfs Editorial Team
  • Aug 03, 2013
    Quiz Created by
    Falmendarez
Cancel
  • All
    All (140)
  • Unanswered
    Unanswered ()
  • Answered
    Answered ()
Oftentimes the use of acronyms facilitates the learning process. What...
Traditionally, matter has been classified into three states
Newspapers and magazines have traditionally been considered what type...
By second grade, students are generally guided to discontinue the...
Identify the steps that best describe the writing process.
What causes earthquakes?
Based on the Individuals with Disabilities Education Act (IDEA),...
Identify the factor(s) that most likely can affect the development of...
The most appropriate exercises for kindergarten and first-grade...
The key function of the food chain in the ecosystem is to
The economy theory that states that prices vary based on balance...
To make the science curriculum cognitively accessible to English...
What are Venn diagrams commonly used for in reading instruction?
Mrs. Morris presented a lesson to her fifth graders about the early...
Identify the order of the products that have supported the Texas...
Why do most countries in the world use the metric system in scientific...
In addition to posting the appropriate rules for laboratory safety,...
The U.S. Constitution was designed so that no single branch of the...
During reading group activity, Ms. Lueck often asks her second graders...
Mrs. Thompson has requested that their students create a PowerPoint...
Scenario: Mr. Jetter organized a lesson to introduce sixth-grade...
What is the primary disadvantage of using written essay to assess the...
Students in Mr. Campos' classroom are learning to interpret visual...
Probably as a response to the war in Iraq and Afghanistan, the...
One of the best known models of scientific inquiry is the 5-E model....
The imperial forces of Japan attacked Pearl Harbor on Sunday, December...
Identify the set of words representing the concept of antonyms
In the problem, , what is the first operation which should be...
The main purpose for teaching sight words is to promote instant...
The process by which physical movements develop and become specialized...
Identify the mathematical properties involved in the following...
Scenario: Mr. Jetter organized a lesson to introduce sixth-grade...
Mike is a four-year-old monolingual, Spanish-speaking child. His...
In a group of people, there are 18 blondes, 17 brunettes, and 5 red...
The Mayflower Compact was one of the earliest agreements to
Perform the indicated operation: (–36) – 11.
Which of the following is LEAST likely to lead to illness and disease?
Which of the following sets of numbers is not an integer followed by...
Identify the statement that best describes the country of Iraq.
How is knowledge of a student's instructional reading level useful...
Identify the statement that best describes the relationship between...
George placed a metal spoon on an open flame and the spoon became so...
Mr. Chapman is a third-grade teacher who is working at a school that...
Mr. Obama uses monosyllabic words to present the concept of onset and...
A rubric to rate the writing skills of children in first grade should...
How many lines of symmetry do all non-square rectangles have?
All of the following are characteristics of emergent readers EXCEPT:
Why might children in prekindergarten and kindergarten have problems...
What is the main function of Earth's atmosphere?
Identify the statement that best represents the rationale for the...
Mr. Rovira guides five-year-old children in the recitation of the...
The English colonies were established in three regions—the New...
Which answer about your metabolism is FALSE?
When teaching English to ELLs of Spanish background, Ms. Rico...
To apply the concept of time zones, students need to have a clear...
There are two categories of maps: reference maps and thematic maps. An...
Stacy went to the store to purchase a loaf of bread. She originally...
A two-year-old child using a crayon to scribble on manila paper is...
A compass rose is a design printed on a chart to
Which of the following sets of symptoms best describes heat stroke?
Running records, teacher observations, and speaking checklists are...
Which of these numbers/number representations are irrational numbers?
Vocabulary development is a key predictor of success in reading....
Alternative assessment, such as verbal reports, laboratory practical...
Convergent research on linear versus curvilinear rhetorical patterns...
After devouring its prey, a crocodile opens its mouth to allow a bird...
The state song of Texas is
The Virginia House of Burgesses was established in Jamestown to
The technical term used to describe the type of vocabulary needed to...
Scenario: Mr. Jetter organized a lesson to introduce sixth-grade...
The most advanced pre-Columbian civilizations of Mesoamerica were the
In music, the words "up" and "down" are usually...
What song has the same rhythm as Happy Birthday?
A card is drawn from a deck of cards, what is the probability that the...
If in the United States we are enjoying a warm winter, what season are...
Germaine, a fourth-grade student, notices that the shape of South...
Vinegar is composed of two substances—acetic acid (a colorless...
In her first-grade class, Ms. Swanson wants to present the fact that...
If a can weighs 14 oz., how many cans would you need to have a ton?...
Which of the following is a locomotor skill?
Xylophones, metallophones, and glockenspiels are instruments...
Identify the informal activities/instruments used to assess oral...
Mrs. Cameron plans to buy carpeting for her living room floor. The...
In what grade does the state curriculum officially introduce children...
Jamie rolls a pair of dice hoping to get an odd number. What is the...
The Sumerians, Akkadians, Babylonians, and Assyrians were some of the...
In the early grades, mathematic and spatial reasoning are introduced...
There are 16 more apples than oranges in a basket of 62 apples and...
Two coins are tossed at the same time. What is the probability that...
The Declaration of Independence consists of a preamble or introduction...
The state curriculum for fine arts and for visual arts is organized...
A globe is a scale model of the Earth shaped like a sphere. A globe...
Identify the main benefit of the shared book experience.
Why are Indian tribes from the Central and Great Plains of Texas...
Vocabulary development is a key predictor of success in reading in the...
Identify the most appropriate strategy(ies) to meet the needs of...
How can teachers minimize the danger of working with chemicals in...
As part of the celebration of Martin Luther King Day, fifth graders...
Two paper bags are each filled with four blue marbles and four red...
An example of a prime number is
In which grade should pupils' experiences in critically evaluating...
Identify the statement that best describes the connection between...
The main purpose of an interactive/dialogue journal is to provide...
Scenario: Corals are invertebrates that generally live in a symbiotic...
Newly fluent readers can read with relative fluency and comprehension....
Identify the strategies that lead the child from the stage of learning...
In a science class for sixth graders, Ms. Anderson posted a list of...
Introducing the multiple versions of stories like Cinderella can help...
Simplify: 6  * 2  +  3  ¸  3
Which of the following words can be used as a good example for...
The use of hot plates and burners are initially allowed in
The Fifteenth Amendment of the U.S. Constitution was ratified in 1870...
Identify the number of syllables and the number of phonemes present in...
Identify the modern-day country that encompasses most of the territory...
Which is not a critical step to follow when solving a problem?
Scenario: Mr. Mackey has seven ELLs in his class who are having...
Simplify to a single term in scientific notation: (2 · 10)3...
The United States is probably the only industrialized country in the...
Scenario: Mr. Mackey has seven ELLs in his class who are having...
Which of the following were the first mathematicians to impact the...
Learning about computers and search engines (A) is a derivative of the...
Which statement best describes an example of the Language Experience...
Characteristics of emergent writers include the following EXCEPT
In 2008 the gross state product of Texas was $1.245 trillion-the...
Marcus can separate a word into its individual phonemes and put it...
What is the main purpose of the Federal Reserve System?
Scenario: Corals are invertebrates that generally live in a symbiotic...
Identify two key historical concepts or characters of the Middle Ages.
Use the following figure to answer the question that follows. Assume...
Living things are classified into five groups: monera, protista,...
What percentage of a daily diet should be composed of carbohydrates?
Which of the following is an example of formative evaluation?
Which of the following is NOT a key element of inquiry-based science...
Which of the following is NOT a developmentally appropriate practice...
Which of the following is equivalent to 17(64 + 82) – 43?
In which grade are the principles of art introduced?
Students who develop an understanding of the key components of a story...
Valerie has a bathtub that she needs to fill with water. To fill the...
During the "engage" component of the 5-E model, students begin the...
Charts, tables, and graphs are commonly used in language arts to
Alert!

Advertisement